Relativistic Kinematics: Distance of B When A Coincides with L

In summary: It's just a convention that I use positive velocities. It's just a convention that I use positive velocities.
  • #1
Amrator
246
83
Homework Statement
A and B both start at the origin and simultaneously head off in opposite directions, each with speed ##\frac{3c}{5}## with respect to the ground. A moves to the right and B moves to the left. Consider a mark on the ground at ##x = L##. As viewed in the ground frame, A and B are a distance ##2L## apart when A passes this mark. As viewed by A, how far away is B when A coincides with the mark?
Relevant Equations
$$t_{observed} = \gamma t_{proper}$$
$$L_{observed} = \frac{L_{proper}} {\gamma}$$
$$t_R - t_L = \frac{Lv}{c^2}$$
$$V = \frac{u+v}{1+\frac{uv}{c^2}}$$
I don't know why I'm so puzzled by this problem; it's only one star. So first, I drew a picture of A and B in the ground frame. Then I drew B and the ground in A's frame. I then used the velocity addition formula to obtain the velocities of both B and the ground relative to A.
$$\frac{v_{B}^{A} + v_{gound}^{A}}{1 + \frac{v_{B}^{A} v_{gound}^{A}}{c^{2}}} = \frac{-\frac{3c}{5} - \frac{3c}{5}}{1 + \frac{9}{25}} = -\frac{15c}{17}$$
And I don't where to go from here.
 

Attachments

  • Problem 1.43.jpg
    Problem 1.43.jpg
    54.2 KB · Views: 139
Last edited:
Physics news on Phys.org
  • #2
What is time reading of A's clock when A is on the mark ? Then multiply it with the velocity you calculated.
 
  • #3
anuttarasammyak said:
What is time reading of A's clock when A is on the mark ? Then multiply it with the velocity you calculated.
I got ##V_{B}^{A}t_{A}^{ground} = \frac{25L}{17}##. So wait, that's the answer? We don't have to take into account time dilation?
 
  • #4
Amrator said:
I got ##V_{B}^{A}t_{A}^{ground} = \frac{25L}{17}##. So wait, that's the answer? We don't have to take into account time dilation?
That can't be right. You must use time from A's frame.
 
  • #5
PeroK said:
That can't be right. You must use time from A's frame.
Ah! Now I see! The mark ##L## is part of the ground frame, so in A's frame, the distance between the origin and ##L## will be contracted! So the time it takes for A to coincide with ##L## in A's frame is $$\Delta t_{A at L} = \frac{\frac{L}{\sqrt{1-\frac{9}{25}}}} {-\frac{3c}{5}}$$. But this gives me a negative time.
 
  • #6
Amrator said:
Ah! Now I see! The mark ##L## is part of the ground frame, so in A's frame, the distance between the origin and ##L## will be contracted! So the time it takes for A to coincide with ##L## in A's frame is $$\Delta t_{A at L} = \frac{\frac{L}{\sqrt{1-\frac{9}{25}}}} {-\frac{3c}{5}}$$. But this gives me a negative time.
I don't understand that calculation.

Why not stick with your original plan to calculate ##v_B\Delta t_A##, where ##v_B## is the speed of ##B## and ##\Delta t_A## is the elapsed time both in ##A##'s frame?
 
  • #7
PeroK said:
I don't understand that calculation.

Why not stick with your original plan to calculate ##v_B\Delta t_A##, where ##v_B## is the speed of ##B## and ##\Delta t_A## is the elapsed time both in ##A##'s frame?
That is what I'm going to do. I'm just trying to get ##\Delta t_A## first. I'm treating the ground in A's frame as almost like a train with proper length ##L##. So then I can calculate the proper time of A ##\Delta t_A## when it coincides with the back of the "train" (i.e ##L##). Is what I'm trying to do nonsense? Please let me know.
 
  • #8
Amrator said:
That is what I'm going to do. I'm just trying to get ##\Delta t_A## first. I'm treating the ground in A's frame as almost like a train with proper length ##L##. So then I can calculate the proper time of A ##\Delta t_A## when it coincides with the back of the "train" (i.e ##L##). Is what I'm trying to do nonsense? Please let me know.
That's fine. In A's frame the origin and the point at ##L## (in the ground frame) are length contracted. Imagine a marker at the origin and another at ##L## in the ground frame. That marker is moving in ##A's## frame and takes ##t_A## to reach ##A##.
 
  • #9
PeroK said:
That's fine. In A's frame the origin and the point at ##L## (in the ground frame) are length contracted. Imagine a marker at the origin and another at ##L## in the ground frame. That marker is moving in ##A's## frame and takes ##t_A## to reach ##A##.
Yup, that's exactly what I'm trying to do. But I'm still getting a negative time from the velocity of the markers in A's frame.
 
  • #10
Amrator said:
Yup, that's exactly what I'm trying to do. But I'm still getting a negative time from the velocity of the markers in A's frame.
That's silly! You have a distance divided by a speed. There should be no negatives involved.
 
  • #11
I'm getting -25L/12c.
 
  • #12
Amrator said:
I'm getting -25L/12c.
What would you do if this was a non-relativistic problem? Would you still get negative time?
 
  • #13
PeroK said:
What would you do if this was a non-relativistic problem? Would you still get negative time?
No, because then I wouldn't be taking into account sign of the velocity. I would just be dividing by the speed. The markers are moving to the left at 3c/5, thus, -3c/5. I then divide ##L/\lambda## by the aforementioned velocity which gives a negative value.
 
  • #14
Amrator said:
No, because then I wouldn't be taking into account sign of the velocity. I would just be dividing by the speed. The markers are moving to the left at 3c/5, thus, -3c/5. I then divide ##L/\lambda## by the aforementioned velocity which gives a negative value.
Why do you only take into account the sign of the velocity because it's SR? You're saying that if the speed is ##10m/s##, then you use ##10m/s## and if it's ##1km/s## you use ##1km/s##, but at a speed of some significant fraction of ##c##, you switch to a negative speed?

At what speed do you switch from from positive to negative speeds?
 
  • #15
If you want to use velocity, which may be negative, you must use displacement, which can also be negative.
 
  • #16
PeroK said:
If you want to use velocity, which may be negative, you must use displacement, which can also be negative.
Yes. Sorry, I confused the frames. ##\Delta t_A## is therefore ##25L/12c##. ##v_{B}^{A} \Delta t_{A} = (-\frac{15c}{17})(\frac{25L}{12c}) = 375L/204##.
 
  • #17
Amrator said:
Yes. Sorry, I confused the frames. ##\Delta t_A## is therefore ##25L/12c##. ##v_{B}^{A} \Delta t_{A} = (-\frac{15c}{17})(\frac{25L}{12c}) = 375L/204##.

That doesn't look right.

What did you get for ##\gamma##?
 
  • #18
PeroK said:
That doesn't look right.

What did you get for ##\gamma##?
I got 5/4.
 
  • #19
Amrator said:
I got 4/5.
You mean ##5/4##?
 
  • #20
PeroK said:
You mean ##5/4##?
Fixed it.
 
  • #21
Amrator said:
Fixed it.
I'll show how I would approach this problem. One option, of course, is to apply the Lorentz Transformation. Alternatively (I'll use ##v## as a fraction of ##c##, sometimes called ##\beta##):

First, in A's frame at ##t_A = 0## the point at ##L## is a distance ##L/\gamma## away. And, moving towards ##A## at speed ##v##. It reaches ##A## after time ##\Delta t_A = \frac{L}{\gamma v}##.

Meanwhile, B has been moving away from ##A## at speed ##v_B = \frac{2v}{1 + v^2}##. Hence is a distance:
$$d = v_B \Delta t_A = \frac{2L}{\gamma(1 + v^2)}$$
Now you can plug in whatever ##v## you've been given.
 
  • Like
Likes Amrator
  • #22
PeroK said:
I'll show how I would approach this problem. One option, of course, is to apply the Lorentz Transformation. Alternatively (I'll use ##v## as a fraction of ##c##, sometimes called ##\beta##):

First, in A's frame at ##t_A = 0## the point at ##L## is a distance ##L/\gamma## away. And, moving towards ##A## at speed ##v##. It reaches ##A## after time ##\Delta t_A = \frac{L}{\gamma v}##.

Meanwhile, B has been moving away from ##A## at speed ##v_B = \frac{2v}{1 + v^2}##. Hence is a distance:
$$d = v_B \Delta t_A = \frac{2L}{\gamma(1 + v^2)}$$
Now you can plug in whatever ##v## you've been given.
Is that gamma associated with the speed of A, B, or ground, and in which frame? That's what I'm confused about right now.
 
  • #23
Amrator said:
Is that gamma associated with the speed of A, B, or ground, and in which frame? That's what I'm confused about right now.
##\gamma## is associated with speed ##v##.
 
  • #24
I'm getting 125L/68. That doesn't seem right.
 
  • #25
Amrator said:
I'm getting 125L/68. That doesn't seem right.
I agree it doesn't look right.

Let me show you one advantage of my method. (I'm very anti plug-and-chug for this reason). I got:
$$d = \frac{2L}{\gamma(1 + v^2)}$$
And I can sanity check this formula. If ##v## is non-relativistic, i.e. ##v \approx 0## and ##\gamma \approx 1##, then we get ##d = 2L##, which is the correct non-relativistic limit.

Also, as ##v \rightarrow c##, we get ##d \rightarrow 0##. Which, if you think about it makes sense.

That doesn't mean my formula is right. But, all you have are numbers that tell you nothing about the physics.

All you have is ##125L/68##, which tells you nothing about the physics of the problem.

In any case, how did you get that number?
 
  • #26
PeroK said:
I agree it doesn't look right.

Let me show you one advantage of my method. (I'm very anti plug-and-chug for this reason). I got:
$$d = \frac{2L}{\gamma(1 + v^2)}$$
And I can sanity check this formula. If ##v## is non-relativistic, i.e. ##v \approx 0## and ##\gamma \approx 1##, then we get ##d = 2L##, which is the correct non-relativistic limit.

Also, as ##v \rightarrow c##, we get ##d \rightarrow 0##. Which, if you think about it makes sense.

That doesn't mean my formula is right. But, all you have are numbers that tell you nothing about the physics.

All you have is ##125L/68##, which tells you nothing about the physics of the problem.

In any case, how did you get that number?
I see.

I actually did the gamma wrong. Now I'm getting 4L/17. I did the following: $$v_{B}\Delta t_{A} = \frac{2L\sqrt{1-\frac{9}{25}}}{1+\frac{9}{25}}$$.
 
  • #27
Amrator said:
I see.

I actually did the gamma wrong. Now I'm getting 4L/17. I did the following: $$v_{B}\Delta t_{A} = \frac{2L\sqrt{1-\frac{9}{25}}}{1+\frac{9}{25}}$$.
$$\frac{2L\sqrt{1-\frac{9}{25}}}{1+\frac{9}{25}} \ne \frac{4L}{17}$$.
 
  • #28
PeroK said:
$$\frac{2L\sqrt{1-\frac{9}{25}}}{1+\frac{9}{25}} \ne \frac{4L}{17}$$.
Right, arithmetic mistake. Now, I'm getting 20L/17.
 
  • Love
Likes PeroK
  • #29
Thank you for the help, @PeroK . I'm going to go back over this to make sure I understand the physics.
 

Related to Relativistic Kinematics: Distance of B When A Coincides with L

1. What is relativistic kinematics?

Relativistic kinematics is a branch of physics that studies the motion of objects at high speeds, close to the speed of light. It incorporates Einstein's theory of relativity to describe how the laws of motion and mechanics change at these extreme velocities.

2. What is the distance of B when A coincides with L?

The distance of B when A coincides with L is known as the proper length. It is the distance measured between two points in a reference frame where both points are at rest relative to the observer. This distance is independent of the observer's motion and is considered the "true" distance between the two points.

3. How does the distance of B change when A moves at high speeds?

As A moves at high speeds, the distance of B will appear to decrease due to the effects of time dilation and length contraction. Time dilation causes time to pass slower for a moving object, while length contraction causes the length of an object to appear shorter in the direction of motion. These effects are a result of the relativity of simultaneity.

4. How is the distance of B affected by the speed of A?

The distance of B is directly affected by the speed of A. As A approaches the speed of light, the distance of B will appear to decrease due to the effects of time dilation and length contraction. At the speed of light, the distance of B will appear to be zero, as there is no time or distance between the two points in a reference frame moving at the speed of light.

5. Can the distance of B ever be greater than the proper length?

No, the distance of B can never be greater than the proper length. This is because the proper length is the "true" distance between two points in a reference frame where both points are at rest relative to the observer. Any other reference frame moving at high speeds will experience the effects of time dilation and length contraction, causing the distance of B to appear shorter.

Similar threads

  • Introductory Physics Homework Help
Replies
1
Views
487
  • Introductory Physics Homework Help
Replies
2
Views
1K
  • Introductory Physics Homework Help
Replies
10
Views
947
  • Introductory Physics Homework Help
Replies
2
Views
347
Replies
11
Views
1K
  • Introductory Physics Homework Help
Replies
1
Views
271
  • Introductory Physics Homework Help
Replies
14
Views
2K
  • Introductory Physics Homework Help
Replies
33
Views
3K
Replies
1
Views
329
Replies
11
Views
2K
Back
Top